subject
Mathematics, 21.03.2020 10:04 Jackie2088

A magazine advertisement claims that wearing a magnetized bracelet will reduce arthritis pain in those who suffer from arthritis. A medical researcher tests this claim with 233 arthritis sufferers randomly assigned to wear either a magnetized bracelet or a placebo bracelet. The researcher records the proportion of each group who report relief from arthritis pain after 6 weeks. After analyzing the data, he fails to reject the null hypothesis. What are valid interpretations of his findings? Which of the following are valid interpretations of his findings? There may be more than one correct answer. Select all answers that apply A. The magnetized bracelets had exactly the same effect as the placebo at reducing arthritis pain. B. The magnetized bracelets are not effective at reducing arthritis pain C. There were no statistically significant differences between the magnetized bracelets and the placebos in reducing arthritis pain. D. There's insufficient evidence that the magnetized bracelets are effective at reducing arthritis pain.

ansver
Answers: 3

Another question on Mathematics

question
Mathematics, 21.06.2019 22:30
(a.s."similarity in right triangles"solve for xa.) 10b.) 20c.) 14.5d.) [tex] 6\sqrt{3} [/tex]
Answers: 1
question
Mathematics, 21.06.2019 22:30
Proving the parallelogram diagonal theoremgiven abcd is a parralelogam, diagnals ac and bd intersect at eprove ae is conruent to ce and be is congruent to de
Answers: 1
question
Mathematics, 21.06.2019 23:00
Solve the system of equations using the substitution method. {y=−3x+7x=−2y−16 enter your answers in the boxes.
Answers: 1
question
Mathematics, 21.06.2019 23:10
You just purchased two coins at a price of $670 each. because one of the coins is more collectible, you believe that its value will increase at a rate of 7.1 percent per year, while you believe the second coin will only increase at 6.5 percent per year. if you are correct, how much more will the first coin be worth in 15 years?
Answers: 2
You know the right answer?
A magazine advertisement claims that wearing a magnetized bracelet will reduce arthritis pain in tho...
Questions
question
Mathematics, 04.07.2019 01:20
question
Mathematics, 04.07.2019 01:20
Questions on the website: 13722363